微觀經濟學

生產函式 q = min {K, L^(1/2)} 的規模報酬是多少?

  • December 17, 2020

我了解到,當規模收益遞減時,平均成本總是在增加。

但教授今天告訴我們,反過來可能並不總是正確的。因此,如果平均成本在增加,這可能並不一定意味著規模回報在減少。

生產函式 $ q = \min {K, L^{(1/2)}} $ 是一個他給我們的東西,作為一個可能的反例來思考。

我們假設資本的投入價格為 2,勞動力的投入價格為 1。

如果我們使用成本最小化來解決這個問題,我們得到 $ C(q) = 2q + q^2 $ 和 $ AC(q) = 2 + q $ 這是一個遞增函式。

如果我輸入 $ kK, kL $ 在生產函式中 $ k > 1 $ ,則函式變為

$ q = \min {kK, (kL)^{1/2}} = \min {kK, k^{1/2} * L^{1/2}}. $

我怎麼知道這顯示的是什麼規模回報?

如果 min 函式很簡單 $ q = \min { K, L } $ ,那麼我可以肯定地說,這顯示了規模收益不變,因為 $ \min {kK, kL} = k*\min{K,L} $ ,但是你如何在數學上解決上述函式的規模回報, $ q = \min {K, L^{1/2}} $ ?

我不知道到達後下一步要採取什麼步驟 $ \min {kK, k^{1/2} * L^{1/2}} $ . 從現在開始,我不知道從 min 函式中提取什麼。

對於如何繼續確定此類範例的規模回報,我將不勝感激。

你有一個 Leontief 生產函式,在最佳狀態下你將永遠擁有 $ K=\sqrt{L}=q_1 $ . 現在按因子增加兩個輸入 $ k>1 $ 你到達 $ kK > \sqrt{kL}=q_2 $ 其中第一個不等式來自 $ k>\sqrt{k} $ 為了 $ k>1 $ 第二個不等式是因為在你的生產函式中只有最小值很重要。因此你有 $ F(kK,kL)= \sqrt{kL}<kK=k\sqrt{L} =kF(K,L) $ 任何最佳輸入組合 $ (K,L) $ 因此規模報酬遞減。

要了解這裡的問題是什麼,請盡職盡責地檢查生產函式中所有可能的子情況。

生產函式為

$$ Q_0 = \min{K_0, L_0^{1/2}} $$.

考慮案例

一種。 $ K_0 < L_0^{1/2} $

這裡 $ Q_0 = K_0 $ . 考慮 $ Q_{\lambda }\equiv \min{\lambda K_0, \lambda^{1/2}L_0^{1/2}},;;; \lambda>1. $

子案例 A1。

如果 $ \lambda K_0\leq \lambda^{1/2}L_0^{1/2} \implies Q_{\lambda} = \lambda K_0 = \lambda Q_0 $ 我們有恆定的規模回報。這發生在 $ K_0 \leq \left(L_0/\lambda\right)^{1/2} $ .

子案例 A2。

如果 $ \lambda K_0 > \lambda^{1/2}L_0^{1/2} \implies Q_{\lambda} = \lambda^{1/2}L_0^{1/2} $ 因為我們在這種情況下 $ K_0\leq L_0^{1/2} $ 和 $ \lambda >1 $ 我們有 $ Q_{\lambda} > Q_0 $ 我們有越來越多的 Reaurns 以擴大規模。這發生在 $ \left(L_0/\lambda\right)^{1/2}< K_0 < L_0^{1/2} $ .


B. $ K_0 > L_0^{1/2} $

這裡 $ Q_0 = L_0^{1/2} $ . 考慮 $ Q_{\lambda }\equiv \min{\lambda K_0, \lambda^{1/2}L_0^{1/2}},;;; \lambda>1. $

在這裡,因為 $ \lambda >1 $ ,我們將永遠擁有 $ \lambda K_0 > \lambda^{1/2}L_0^{1/2} \implies Q_{\lambda} = \lambda^{1/2}L_0^{1/2} = \lambda^{1/2} Q_0 $ 所以我們得到規模收益遞減。

C。 $ K_0 = L_0^{1/2} $

這裡也是,因為 $ \lambda >1 $ ,我們處於與案例 B 相同的情況,因此規模收益遞減。


所以看起來“出發點”(輸入情況為 $ Q_0 $ ) 很重要——這就是為什麼在另一個答案中呼叫了最佳行為,以便在某種有意義的意義上確定這個“出發點”。

引用自:https://economics.stackexchange.com/questions/41629